Jump to content

Secrets In Inequalities VP's Content

There have been 298 items by Secrets In Inequalities VP (Search limited from 26-05-2020)



Sort by                Order  

#382021 $k!+48=48(k+1)^{m}$

Posted by Secrets In Inequalities VP on 30-12-2012 - 17:17 in Số học

Có thiếu đk $k+1$ nguyên tố không nhỉ?

$k+1$ nguyên tố chỉ là $1$ TH của bài này thôi cậu .
Nếu $k+1$ là hợp số $\Rightarrow k+1= x.y$ ( $x,y\leq k$ )
$\Rightarrow k!= 1.2...x...y...k\vdots xy\Rightarrow k!\vdots k+1\Rightarrow 48\vdots k+1$.Dễ rồi !



#381865 chứng minh rằng hoặc p2 | a + b, hoặc p3 | a3 + b3

Posted by Secrets In Inequalities VP on 30-12-2012 - 08:16 in Số học

Cho p > 3 là số nguyên tố. Giả sử a, b là các số nguyên sao cho p | a + b và p2 | a3 + b3. Hãy chứng minh rằng hoặc p2 | a + b, hoặc p3 | a3 + b3

Vì $p^{2} | a ^{3}+ b^{3}= (a+b)(a^2-ab+b^2)$ nên chỉ xảy ra
+ $p^{2} | (a+b)$ ta có đpcm
+ $p | a^2-ab+b^2$ .Mà $p | a+b\Rightarrow p | (a+b)^2= a^2+2ab+b^2$
$\Rightarrow p | 3ab$ .Mà $p>3$ $\Rightarrow (p,3)=1\Rightarrow p | ab$
Lại có $p | a+b,p \in \mathbb{P}$ nên $p | a$ và $p | b$
$\Rightarrow p^{3} | a^{3}, p^{3} | b^{3}\Rightarrow p^{3} | a^{3}+b^3$



#381768 $(n+1)^n-1 \vdots n^2$

Posted by Secrets In Inequalities VP on 29-12-2012 - 22:12 in Các dạng toán THPT khác

2.Cho $a\geq 3; n> 1$ và thoả mãn $a^{n}\equiv 1 (mod n)$ với n là số tự nhiên. Gọi p là ước nguyên tố nhỏ nhất của n.
CMR : $a-1\vdots p$

Ta có :$a^{n}\equiv 1(Mod n)\Rightarrow a^{n}\equiv 1(Mod p)$
Gọi $h$ là cấp của $a$ theo mod $p$ $\Rightarrow a^{h}\equiv 1(Mod p)$
Theo tính chất của cấp thì $h|n$
Theo Fermart thì $h|p-1$
Ta thấy $h=1$ và nếu $h\neq 1$ thì gọi $q$ là ước nguyên tố nhỏ nhất của $h$ $\Rightarrow q|h\Rightarrow q|n$
Mà $q\leq h\leq p-1< p$ nên mâu thuẫn với việc $p$ là ước nguyên tố nhỏ nhất của $n$
Suy ra $h=1$ $\Rightarrow a\equiv 1(Modp)\Rightarrow a-1\vdots p$



#381696 $$3\left( {{x}^{2}}+{{...

Posted by Secrets In Inequalities VP on 29-12-2012 - 20:16 in Bất đẳng thức và cực trị

Cho $x,y,z$ là các số dương. Chứng minh rằng với $x+y+z=3$ thì
$$3\left( {{x}^{2}}+{{y}^{2}}+{{z}^{2}} \right)+4xyz\ge 13$$

Làm thế này cho đỡ đau đầu !
Ta thấy sẽ có ít nhất 2 trong 3 số $x,y,z$ cùng lớn hơn hoặc nhỏ hơn $1$.Giả sử là $y$ và $z$
$\Rightarrow (y-1)(z-1)\geq 0\Leftrightarrow yz\geq y+z-1= 2-x$
$\Rightarrow xyz\geq 2x-x^2\Rightarrow 4xyz\geq 8x-4x^2$
Áp dụng nhận xét trên kết hợp vs bđt cơ bản $a^2+b^2\geq \frac{(a+b)^2}{2}$ ta được :
$VT\geq 3x^2+\frac{3}{2}.(y+z)^2+8x-4x^2= \frac{3}{2}.(3-x)^2-x^2+8x$
$$= \frac{x^2}{2}-x+\frac{1}{2}+13= \frac{(x-1)^2}{2}+13\geq 13$$
Vậy ta có $Q.E.D$



#381566 Chứng minh tồn tại $k_1,k_2,...,k_s$ để $b_1+m_1k_1=b_2+m_2k_2...

Posted by Secrets In Inequalities VP on 29-12-2012 - 14:37 in Số học

Cho $m_1,m_2,...,m_s$ là các số tự nhiên thỏa $(m_i,m_j)=1$ với mọi $i \ne j$. Cho $b_1,b_2,...,b_s \in \mathbb{Z}$. Chứng minh tồn tại $k_1,k_2,...,k_s \in \mathbb{Z}$ sao cho:
$$b_1+m_1k_1=b_2+m_2k_2=...=b_s+m_sk_s$$

Xét hệ phương trình đồng dư : $x\equiv {b_i} (Mod{m_i})$
Theo định lí thặng dư của Tàu thì hệ luôn có nghiệm $c$.$\Rightarrow c= {b_i}+{m_i}{k_i}$ suy ra $Q.E.D$



#381559 Chứng minh $m\vdots 1979$

Posted by Secrets In Inequalities VP on 29-12-2012 - 14:11 in Số học

Cho m,n là các số nguyên dương sao cho:
$\frac{m}{n}=1-\frac{1}{2}+\frac{1}{3}-\frac{1}{4}+...-\frac{1}{1318}+\frac{1}{1319}$
Chứng minh $m\vdots 1979$

Ta thấy $1979$ là số nguyên tố .
Ta có : $\frac{m}{n}=1-\frac{1}{2}+\frac{1}{3}-\frac{1}{4}+...-\frac{1}{1318}+\frac{1}{1319}$
$$=(1+\frac{1}{2}+\frac{1}{3}+\frac{1}{4}+...+\frac{1}{1318}+\frac{1}{1319})-2.(\frac{1}{2}+\frac{1}{4}+...+\frac{1}{1318})$$
$$=(1+\frac{1}{2}+\frac{1}{3}+\frac{1}{4}+...+\frac{1}{1318}+\frac{1}{1319})-(1+\frac{1}{2}+...+\frac{1}{659})$$
$$=\frac{1}{660}+\frac{1}{661}...+\frac{1}{1318}+\frac{1}{1319}$$
$$=(\frac{1}{660}+\frac{1}{1319})+(\frac{1}{661}+\frac{1}{1338})+...$$
$$=\frac{1979}{660.1319}+\frac{1979}{661.1318}+...$$
$$=1979.(\frac{1}{660.1319}+\frac{1}{661.1318}+...)= 1979.\frac{p}{q}$$
trong đó $q=660.661...1319\Rightarrow (q,1979)=1$
Ta có : $\frac{m}{n}= 1979.\frac{p}{q}\Rightarrow mq=1979np\Rightarrow mq\vdots 1979$
Mà $(q,1979)=1\Rightarrow m\vdots 1979\Rightarrow Q.E.D$



#381558 Với mỗi số nguyên dương n, ký hiệu S(n) là tổng tất cả các chữ số trong biểu...

Posted by Secrets In Inequalities VP on 29-12-2012 - 13:55 in Số học

Với mỗi số nguyên dương n, ký hiệu S(n) là tổng tất cả các chữ số trong biểu diễn thập phân của n.
Xét các số nguyên dương m là bội của 2003. Hãy tìm giá trị nhỏ nhất của S(m).

Đặt $p=2003\in\mathbb{P}$.
+ Nếu $S(n)= 1\Rightarrow n= 100...000= 10^{^{k}}\not\vdots 2003\Rightarrow S(n)> 1$
+ Nếu $S(n)= 2$$\Rightarrow n= 200...000$ hoặc $n=100...100...00= 10^{i}+10^{j}$
Ta có $n=200...000= 2.10^{k} \not\vdots 2003$ $\Rightarrow n=10^i+10^j$
$\Rightarrow 10^i+10^j\vdots 2003=p\Rightarrow 10^i= -10^j(Modp)$
$\Rightarrow 10^{i-j}= -1(Modp)$$\Rightarrow 10^k= -1(Modp)$ ( $k=i-j$ )
Dễ thấy $2^{10}= 1024\equiv 10^7(Modp)$ , suy ra :
$(2^{5k})^{2}= 2^{10k}\equiv (10^{k})^{7}\equiv -1(Modp)$
$\Rightarrow -1$ là số chính phương mod p suy ra $p$ có dạng $4k+1$
$\Rightarrow 2003=p= 4k+1\Rightarrow k= \frac{2002}{4} \not \in \mathbb{Z}$
Suy ra vô lí $\Rightarrow S(n)> 2$
+ Bây giờ ta chứng minh là có $n$ để $S(n)= 3$
Vì $p=2003\neq 8k-1\neq 8k+1$ suy ra $2$ không là SCP mod p $\Rightarrow 2^{\frac{p-1}{2}}\equiv -1(Modp))$
Mà $10^{7}\equiv 2^{10}(Modp)\Rightarrow 2.10^{700}\equiv 2^{1001}= 2^{\frac{p-1}{2}}\equiv -1 (Modp)$
$\Rightarrow (2.10^{700}+1)\vdots p$ .Mà $S(2.10^{700}+1)= 3$ nên khẳng định của ta là đúng.
Vậy $minS(n)= 3$.Xảy ra khi chẳng hạn $n= 2.10^{700}+1$



#379636 Cho $x,y,z>0$. CMR: $\sum\dfrac{2x^2+xy...

Posted by Secrets In Inequalities VP on 22-12-2012 - 20:27 in Bất đẳng thức và cực trị

Chuẩn hóa $x + y + z = 1$
Ta có $$\left (y + \sqrt{xz} + z\right )^2 \le \left (y + x + z\right )\left (y + z + z\right ) = (x + y + z)(y + 2z) = y + 2z$$
Nên $$VT \ge \dfrac{2x^2 + xy}{y + 2z} + \dfrac{2y^2 + yz}{z + 2x} + \dfrac{2z^2 + zx}{x + 2y} $$
Lại có $$\dfrac{2x^2 + xy}{y + 2z} + x(2x + y).(y + 2z) \ge 2\left (2x^2 + xy\right ) \Leftrightarrow \dfrac{2x^2 + xy}{y + 2z} \ge 2\left (2x^2 + xy\right ) - x(2x + y)(y + 2z) \ge 2\left (2x^2 + xy\right )$$ $$ - \dfrac{x(2x + 2y + 2z)^2}{4} = 2\left (2x^2 + xy\right ) - x$$
Từ đó, ta có :
$$VT \ge 2\left (\left (2x^2 + xy\right ) + \left (2y^2 + yz\right ) + \left (2z^2 + zx\right )\right ) - \left (x + y + z\right ) = 3\left (x^2 + y^2 + z^2\right )$$ $$ + \left (x + y + z\right )^2 - 1 \ge (x + y + z)^2 + 1 - 1 = 1$$
Suy ra ĐPCM.



#379628 $\sum \frac{1}{(a+2b)^2}\geq \fr...

Posted by Secrets In Inequalities VP on 22-12-2012 - 20:11 in Bất đẳng thức và cực trị

Bài 2/ Chứng minh rằng với mọi $a,b,c>0$
$\frac{1}{b^2+bc+c^2}+\frac{1}{c^2+ca+a^2}+\frac{1}{a^2+ab+b^2}\geq \frac{9}{(a+b+c)^2}$
(Cũng đã lâu rồi nhưng cũng rất hay, @Đạt: không cần chuẩn hóa và rất cổ điển)

Ta có : $\frac{1}{b^2+bc+c^2}= \frac{ab+bc+ca}{(b^2+bc+c^2)(ab+bc+ca)}\geq \frac{4(ab+bc+ca)}{(b^2+bc+c^2+ab+bc+ca)^2}$
$$= \frac{4(ab+bc+ca)}{(b+c)^2(a+b+c)^2}$$
Tương tự rồi cộng vào ta cần chứng minh :
$\frac{4(ab+bc+ca)}{(b+c)^2(a+b+c)^2}+\frac{4(ab+bc+ca)}{(c+a)^2(a+b+c)^2}+\frac{4(ab+bc+ca)}{(a+b)^2(a+b+c)^2}\geq \frac{9}{(a+b+c)^2}$
$\Leftrightarrow \frac{1}{(b+c)^2}+\frac{1}{(c+a)^2}+\frac{1}{(a+b)^2}\geq \frac{9}{4(ab+bc+ca)}$
Đúng theo $IRan96$



#379624 $\frac{1}{(a+1)^2}+\frac{1}...

Posted by Secrets In Inequalities VP on 22-12-2012 - 19:59 in Bất đẳng thức - Cực trị

Đề thi học kỳ bài khó thứ nhì trường THPT chuyên Hà Nội-Amsterdam Khá quen thuộc.
Bài toán: Cho$a,b,c$ là các số thực dương $abc=1.$. Chứng minh rằng:

$\frac{1}{(a+1)^2}+\frac{1}{(b+1)^2}+\frac{1}{(c+1)^2}+\frac{2}{(a+1)(b+1)(c+1)}\geq 1$

Ta thấy trong 3 số $a,b,c$ sẽ có 2 số cùng lớn hơn hoặc nhỏ hơn 1.Giả sử là $a$ và $b$.
$(a-1)(b-1)\geq 0\Leftrightarrow ab+1\geq a+b\Leftrightarrow 2(ab+1)\geq (a+1)(b+1)$
Áp dụng nhận xét trên kết hợp vs bổ đề qen thuộc $\frac{1}{(x+1)^2}+\frac{1}{(y+1)^2}\geq \frac{1}{xy+1}$ :
$VT\geq \frac{1}{ab+1}+\frac{1}{(c+1)^2}+\frac{2}{2(ab+1)(c+1)}= \frac{1}{\frac{1}{c}+1}+\frac{1}{(c+1)^2}+\frac{1}{(\frac{1}{c}+1)(c+1)}$
$$= \frac{c}{c+1}+\frac{1}{(c+1)^2}+ \frac{c}{(c+1)^2}$$
$$= \frac{c(c+1)}{(c+1)^2}+\frac{1}{(c+1)^2}+ \frac{c}{(c+1)^2}= \frac{(c+1)^2}{(c+1)^2}= 1$$
-------------
Hỵ hỵ hôm qua cũng nghĩ ra thêm cách này mà lười p0st :3 :luoi:



#378398 CMR: $\triangle ABC$ đều

Posted by Secrets In Inequalities VP on 17-12-2012 - 21:16 in Hình học

Bài toán: Chứng minh rằng, tam giác $ABC$ đều khi và chỉ khi
$$\frac{a\cdot\angle A + b\cdot\angle B + c\cdot\angle C}{a+b+c} = 60^\circ$$

Giả sử $a\geq b\geq c\Rightarrow \angle A\geq \angle B\geq \angle C$
Theo $chebyshev$ : $\frac{a\cdot\angle A + b\cdot\angle B + c\cdot\angle C}{a+b+c} \geq \frac{(a+b+c)(\angle A+\angle B+\angle B)}{3(a+b+c)}= 60$
Xét dấu bằng một chút là ta có $Q.E.D$



#375996 Chứng minh rằng tồn tại vô số n để $a_n$ chia hết cho 7

Posted by Secrets In Inequalities VP on 08-12-2012 - 14:28 in Dãy số - Giới hạn

Cho $\large{(a_n)\begin{cases}a_0=1\\a_n=a_{n-1}+a_{[\frac{n}{2}]}\end{cases}}$
Chứng minh rằng tồn tại vô số n để $a_n$ chia hết cho 7

Giả sử chỉ có hữu hạn số chia hết cho $7$ và $a_k$ là số cuối cùng trong các số ấy
Theo ct xác định dãy ta có : $a_{2k+1}= a_{2k}+a_k;a_{2k}= a_{2k-1}+a_k$
$\Rightarrow a_{2k-1}\equiv a_{2k}\equiv a_{2k+1}\equiv b \pmod 7$
Ta có : $a_{4k-3}\equiv a_{4k-3}+0.b \pmod 7$
$a_{4k-2}= a_{4k-3}+a_{2k-1}\equiv a_{4k-3}+1.b \pmod 7$
$a_{4k-1}= a_{4k-2}+a_{2k-1}\equiv a_{4k-3}+2.b \pmod 7$
$a_{4k}= a_{4k-1}+a_{2k}\equiv a_{4k-3}+3.b \pmod 7$
$a_{4k+1}= a_{4k}+a_{2k}\equiv a_{4k-3}+4.b \pmod 7$
$a_{4k+2}= a_{4k+1}+a_{2k+1} \equiv a_{4k-3}+5.b \pmod 7$
$a_{4k+3}= a_{4k+2}+a_{2k+1} \equiv a_{4k-3}+6.b \pmod 7$
Do $\gcd (b,7)=1$ nên các số $a_{4k-3}+ib$ với $i$ chạy từ $0$ đến $6$ lập thành 1 HĐĐ modulo 7.
Nên trong $7$ số $a_{4k-3}+ib$ phải có một số chia hết cho $7$, mà số này lại lớn hơn $a_k$: mâu thuẫn vs điều giả sử
Vậy giả sử là sai và ta có $Q.E.D$

p/s :Ai sửa hộ mình cái Latex vs .Thanks trước !

Perfectstrong: Đề nghị em nên học một lớp bổ túc về latex gấp :P



#375640 Chứng minh rằng phương trình $x^{4}+3y^{4}=z^{4...

Posted by Secrets In Inequalities VP on 06-12-2012 - 20:38 in Số học

Chứng minh rằng phương trình $x^{4}+3y^{4}=z^{4}$ có vô số nghiệm nguyên

Ta thấy : $k^{4}+3.0^{4}= k^4$
Vậy pt có vô số nghiệm nguyên $(x,y,z)=(k,0,k)$ với $k\in\mathbb{Z}$



#375309 Xác định giá trị $p,q$ để $S$ là một số nguyên tố

Posted by Secrets In Inequalities VP on 05-12-2012 - 15:19 in Số học

Ta có nhận xét là nếu $a \not\in\mathbb{Z}$ thì $[a]+[-a]= -1$
Xét : $[\frac{kq}{p}]+[\frac{(p-k)q}{q}]= [\frac{kq}{p}]+[q-\frac{kp}{q}]= q+[\frac{kq}{p}]+[-\frac{kq}{p}]$ $= q-1$
( vì do $(p,q)=1$ nên $\frac{kp}{q}\not\in\mathbb{Z}$ nên áp dụng NX trên thì ta có đc điều này )
Tương tự suy ra : $S= \frac{(p-1)(q-1)}{2}$
Dễ rồi !



#375185 Tìm tất cả các cấp số cộng có vô hạn số hạng sao cho tồn tại số $N$...

Posted by Secrets In Inequalities VP on 04-12-2012 - 21:25 in Số học

Tìm tất cả các cấp số cộng có vô hạn số hạng sao cho tồn tại số $N$ mà với mọi $p>N$, nếu $a_p$ nguyên tố thì $p$ cũng là số nguyên tố



#374340 $\frac{a}{a^2+ab+bc}+\frac{b}...

Posted by Secrets In Inequalities VP on 01-12-2012 - 21:05 in Bất đẳng thức và cực trị

Bài 6: Cho các số thực $a,b,c>0$ sao cho $abc=1$, chứng minh rằng: $$(a+b)(b+c)(c+a)\geq 4(a+b+c-1)$$

Thêm 1 bài cuối nhé, tờ giấy bị rách câu 6 mới tìm lại được :icon6:

Lâu lâu chém tí BĐT cho vui ! ^^
Áp dụng bổ đề quen thuộc :$(a+b)(b+c)(c+a)\geq \frac{8}{9}(ab+bc+ca)(a+b+c)$
Ta chỉ cần phải CM : $\frac{8}{9}(ab+bc+ca)(a+b+c)\geq 4(a+b+c-1)$
Chia cả 2 vế cho $a+b+c$ ta đc BĐT tương đương là :
$$\frac{8}{9}(ab+bc+ca)+\frac{4}{a+b+c}\geq 4$$
Đúng theo $AM-GM$ :
$VT= \frac{4}{9}(ab+bc+ca)+\frac{4}{9}(ab+bc+ca)+\frac{4}{a+b+c}\geq 3\sqrt[3]{\frac{4^{3}}{9^{2}}\frac{(ab+bc+ca)^2}{a+b+c}}$
$$3\sqrt[3]{\frac{4^{3}}{9^{2}}\frac{(ab+bc+ca)^2}{a+b+c}}\geq 3\sqrt[3]{\frac{4^{3}}{9^{2}}.\frac{3abc(a+b+c)}{a+b+c}}= 4$$
Vậy ta có $Q.E.D$



#374311 Chứng minh rằng có vô số số nguyên tố dạng: $p=8k+1$

Posted by Secrets In Inequalities VP on 01-12-2012 - 20:27 in Số học

Chứng minh rằng có vô số số nguyên tố dạng:
a) $p=8k+1$

Tổng quát : Với mỗi số n nguyên dương cho trước luôn tồn tại vô số số nguyên tố dạng $2^{n}k+1$
Ta dùng bổ đề :
Bổ đề : Cho $a,n\in\mathbb{N}^{*}$ .Khi đó mọi ước nguyên tố p của số $a^{2^{n}}+1$ hoặc bằng $2$ hoặc có dạng $p= 2^{n+1}k+1$
Vào bài toán :
Giả sử chỉ có hữu hạn số ntố dạng $2^{n}k+1$ là ${p_1},{p_2},..,{p_m}$
Đặt $a=2{p_1}{p_2}...{p_m}$
Xét số $A= a^{2^{n-1}}+1$ .Theo bổ đề thì mọi ước ntố p của $A$ đều có dạng $p= 2^{n}k+1$.
Số $p$ này phải khác tất cả các ${p_i}$ vì nếu $p= {p_i}$ nào đó thì $a\vdots p$ mà $A\vdots p$ $\Rightarrow 1\vdots p$ vô lí
$\Rightarrow p\neq {p_i}$
Mà $p= 2^{n}k+1$. nên mâu thuẫn vs giả sử $\Rightarrow$ giả sử sai $\Rightarrow Q.E.D$



#372827 $x^{2009}+y^{2009}=7^{z}$

Posted by Secrets In Inequalities VP on 26-11-2012 - 19:37 in Số học

Tìm x,y,z tự nhiên thỏa mãn:
$x^{2009}+y^{2009}=7^{z}$

Vì $2009= 7^{2}.41$ và $x+y|x^{2009}+y^{2009}$ nên dễ thấy $7|x+y$
Theo LTE : ${v_p}(x^{2009}+y^{2009})= {v_p}(x+y)+{v_p}(2009)= {v_p}(x+y)+2$
$\Rightarrow x^{2009}+y^{2009}= 49k(x+y)$
Mà $x^{2009}+y^{2009}=7^{z}$ là luỹ thua của 7 nên suy ra $k=1$
$\Rightarrow x^{2009}+y^{2009}= 49(x+y)$
Dễ thấy rằng nếu $(x,y)\neq (1,1)$ thì $VT> VP$
Lại dễ thấy $(1,1)$ không thỏa mãn đễ nên pt vô nghiệm !



#372364 $a^{2^{n}}+b^{2^{n}}$

Posted by Secrets In Inequalities VP on 25-11-2012 - 10:33 in Số học

Giải trí nhân ngày 26/11 (ngày mai ) :P ! Cũng dễ thôi !
Cho $a,b \in\mathbb{N}^{*}$ , $(a,b)=1$. CMR : Mọi uoc nguyên tố lẻ của$a^{2^{n}}+b^{2^{n}}$ đều có dạng $2^{n+1}k+1$



#372357 $a^{3}b^{3}+b^{3}c^{3}+c^{3...

Posted by Secrets In Inequalities VP on 25-11-2012 - 10:21 in Bất đẳng thức - Cực trị

Cho các số không âm a,b,c thỏa a+b+c=2. Chứng minh rằng:
$a^{3}b^{3}+b^{3}c^{3}+c^{3}a^{3}\leq 1$

Lâu lâu ms làm bđt ! :biggrin:
Giả sủ $a=max(a,b,c)$
Dễ thấy $3a^3(b+c)\geq b^2c^2\Rightarrow 3a^3bc(b+c)\geq b^3c^3$
$\Rightarrow VT\leq a^3b^3+3a^3bc(b+c)+a^3c^3= a^3(b+c)^3$
$a(b+c)\leq \frac{[[a+(b+c)]]^2}{4}= 1\Rightarrow VT\leq 1\Rightarrow Q.E.D$



#372155 $\widehat {BOC} = \widehat {AOD}$

Posted by Secrets In Inequalities VP on 24-11-2012 - 19:59 in Hình học

Cho tứ giác lồi ABCD.Gọi $E = AB \cap CD;F = AD \cap BC;P = AC \cap BD$.GỌI O là hình chiếu của P trên EF.CMR$\widehat {BOC} = \widehat {AOD}$

+ TH1: $BD//EF$ dễ thấy $Q.E.D$

+ TH2: $BD$ không song song với $EF$

$S= BC\cap EF;K=BP\cap AF$

$\Rightarrow (ADKF)= -1\Rightarrow (ACPS)= -1$ ( chiếu xuyên tâm E )

$\Rightarrow O(ACPS)= -1$.

$OP \bot OS \Rightarrow \widehat{POA} = \widehat{POC}$

Ta có : $(ADKF)= -1 \Rightarrow E(ADKF)= -1 \Rightarrow E(BDPF)= -1 \Rightarrow O(BDPF)= -1$.

$OP \bot OF \Rightarrow \widehat{POB} = \widehat{POF}$

Kết hop 2 cái màu đỏ suy ra :

$\widehat{BOA}= \widehat{DOC} \Rightarrow \widehat{BOC}=\widehat{DOA}= \widehat{AOD} \Rightarrow Q.E.D$

p/s: thanks a Hân ! :lol:



#371838 CMR: $AA_{1}=BA_{1}$

Posted by Secrets In Inequalities VP on 23-11-2012 - 19:01 in Hình học

Bài toán
Cho các điểm$A_{1},B_{_{1}}$ theo thứ tự thuộc CB,CA của tam giác ABC. D=$A_{1}A\cap BB_{_{1}}=D;A_{1}B_{1}\cap CD=E.$ Biết rằng $\measuredangle A_{1}EC=90$ và $A,B,E,A_{1}$ cùng thuộc 1 đường tròn. CMR: $AA_{1}=BA_{1}$


$AE\cap BC= F$.$\Rightarrow (BFAC)= -1\Rightarrow E(BFAC)= -1$
Mà $EA\perp EC\Rightarrow \angle AEB= \angle AEF\Rightarrow \angle {A_1}BA= \angle AEF= \angle AEB={A_1}AB $
$\Rightarrow \Delta A{A_1}B $ cân tại ${A_1}$ $\Rightarrow A{A_1}= B{A_1}$



#371033 Tính tổng $\sum_{2012}^{4024}p\left ( n...

Posted by Secrets In Inequalities VP on 20-11-2012 - 20:21 in Số học

Với mỗi số tự nhiên $n$, gọi $p(n)$ là ước số lẻ lớn nhất của $n$. Hãy tính tổng $\sum_{n=2012}^{4024}p\left ( n \right )$.

Đặt $v(n)$ là số mũ cao nhất của $2$ trong $n$ . Chú ý $v(n)p(n)= n\Rightarrow p(n)= \frac{n}{v(n)}$ là ra !



#369761 $\sum _{0\leq k\leq n;(k,n)=1}k= \frac...

Posted by Secrets In Inequalities VP on 15-11-2012 - 22:27 in Số học

Cho $n$ là số tự nhiên lớn hơn 1 . Chứng minh rằng : $\sum _{0\leq k\leq n;(k,n)=1}k= \frac{1}{2}n\phi (n)$

Bài này chỉ cần một nhận xét nhỏ là solved problem :P :
Dễ thấy nếu $gcd(k,n)= 1$ thì $gcd(n,n-k)= 1$
$\Rightarrow \sum _{0\leq k\leq n;(k,n)=1}k= \sum _{0\leq k\leq n;(k,n)=1}(n-k)$$\Rightarrow \sum _{0\leq k\leq n;(k,n)=1}k= \frac{1}{2}( \sum _{0\leq k\leq n;(k,n)=1}k+ \sum _{0\leq k\leq n;(k,n)=1}(n-k))= \frac{1}{2}\phi (n)n$
( vì số các số k mà $gcd(k,n)=1$ bằng $\phi (n)$ )



#369746 $S = C_{2n + 1}^0{.2^{2n}} + C_{2n +...

Posted by Secrets In Inequalities VP on 15-11-2012 - 21:56 in Số học

Đặt $a=1+\sqrt{3},b= 1-\sqrt{3}\Rightarrow ab=-2;\frac{a^2}{2}= 2+\sqrt{3};\frac{b^2}{2}= 2-\sqrt{3}$
${P_n}= \frac{1}{2}(a^{2n+1}+b^{2n+1})$
Dùng khai triển Newton cho $(1+\sqrt{3})^{2n+1},(1-\sqrt{3})^{2n+1}$ta suy ra đc ${P_n}= \sum_{k=0}^{n}\binom{2k}{2n+1}.3^k$ là số nguyên .
Lại dùng Newton cho $(2+\sqrt{3})^{2n+1},(2-\sqrt{3})^{2n+1}$ ta đc:
${S_n}= \frac{(\frac{a^2}{2})^{2n+1}+(\frac{b^2}{2})^{2n+1}}{4}= \frac{a^{4n+2}+b^{4n+2}}{2^{2n+3}}= \frac{a^{4n+2}+2(ab)^{2n+1}+b^{4n+2}}{2^{2n+3}}+\frac{1}{2}= \frac{{P_n}^{2}}{2^{2n+1}}+\frac{1}{2}$
Nhân chéo lên
$\Rightarrow {P_n}^{2}\vdots 2^{2n},{P_n}^{2}\not\vdots 2^{2n+1}\Rightarrow {P_n}^{2}= m2^{2n}$ ( m lẻ )
Mà ${P_n}^{2}$ và $2^{2n}$ chính phuong nên $m= (2p+1)^{2}$
$\Rightarrow {P_n}^{2}= (2p+1)^22^{2n}$
$\Rightarrow {P_n}= (2p+1)2^{^{n}}\Rightarrow\frac{{P_n}-2^n}{2^{n+1}}= p$
Mà dễ thấy :
${S_n}= \frac{{P_n}^{2}}{2^{2n+1}}+\frac{1}{2}= (\frac{{P_n}-2^n}{2^{n+1}})^{2}+(\frac{{P_n}+2^n}{2^{n+1}})^{2}= p^{2}+(p+1)^{2}$
Vậy ta có $Q.E.D$